Module 4 Exam Flashcards

1
Q

With reference to the ligaments of the lower extremity joints, which statement is correct?

A

The lateral collateral ligaments of the ankle are most often sprained partially due to natural inversion at the subtalar joint

How well did you know this?
1
Not at all
2
3
4
5
Perfectly
2
Q

When the hip flexes in standing, the following motions are likely to occur.

A

knee flexion and ankle dorsiflexion

How well did you know this?
1
Not at all
2
3
4
5
Perfectly
3
Q

Which of the following correctly describes the bones that make up the hip joint?

A

acetabulum (ilium, ischium, pubis) and proximal femur

How well did you know this?
1
Not at all
2
3
4
5
Perfectly
4
Q

In upright standing, posterior pelvic tilt is associated with:

A

hip extension; decrease in lumbar lordosis

How well did you know this?
1
Not at all
2
3
4
5
Perfectly
5
Q

____________ is a hip deformity commonly seen in patients with cerebral palsy, that predisposes the person to hip dislocation because the femoral neck angle of inclination is increased.

A

Coxa valga

How well did you know this?
1
Not at all
2
3
4
5
Perfectly
6
Q

Three muscles that insert on the pes anserine are _______________.

A

sartorius, gracilis and semitendinosus

How well did you know this?
1
Not at all
2
3
4
5
Perfectly
7
Q

During gait, what is the gluteus medius responsible for?

A

Reducing drop of the opposite side of the pelvis

How well did you know this?
1
Not at all
2
3
4
5
Perfectly
8
Q

The two obturator muscles ______________________.

A

both externally rotate the hip

How well did you know this?
1
Not at all
2
3
4
5
Perfectly
9
Q

_________________ is the muscle that unlocks the knee.

A

Popliteus

How well did you know this?
1
Not at all
2
3
4
5
Perfectly
10
Q

The knee joint__________________.

A

is dependent on ligaments and muscles for stability

How well did you know this?
1
Not at all
2
3
4
5
Perfectly
11
Q

Which statement is most correct with regard to the Q angle of the knee?

A

Excessive Q angle increases the likelihood of patellar dysfunction

How well did you know this?
1
Not at all
2
3
4
5
Perfectly
12
Q

Which statement is most correct, with regard to coxa varum?

A

Coxa varum is a reduced angle of inclination at the femoral neck

How well did you know this?
1
Not at all
2
3
4
5
Perfectly
13
Q

What effect will removal of the patella have?

A

Reduce the functional strength of the quadriceps by 50%

How well did you know this?
1
Not at all
2
3
4
5
Perfectly
14
Q

The anterior cruciate ligament _______________________.

A

attaches the anterior tibia to the posterior femur, protecting the tibia from forward dislocation

How well did you know this?
1
Not at all
2
3
4
5
Perfectly
15
Q

Genu valgum is also referred to as ___________________ and can result in excessive tensile forces on the _______________ ligament of the knee.

A

knock knees/medial collateral

How well did you know this?
1
Not at all
2
3
4
5
Perfectly
16
Q

With reference to the patellofemoral joint:

A

Excessive lateral patellar tracking occurs with increased Q-angle.

How well did you know this?
1
Not at all
2
3
4
5
Perfectly
17
Q

With regard to the tibiofemoral joint, which answer is MOST correct?:

A

The joint is stabilized laterally by the lateral collateral ligament, IT band, and biceps femoris muscle

How well did you know this?
1
Not at all
2
3
4
5
Perfectly
18
Q

During the “screw home” mechanism of the knee joint which of the following occurs?

A

The medial femoral condyle slides backward as the knee moves into full extension

How well did you know this?
1
Not at all
2
3
4
5
Perfectly
19
Q

Which muscles are found on the medial and posteromedial side of the knee?

A

Sartorius, gracilis, semitendinosus, semimembranosus

How well did you know this?
1
Not at all
2
3
4
5
Perfectly
20
Q

With regard to the quadricep muscles, which statement is the most correct?

A

They allow the patella to float above the femur when the knee nears full extension

How well did you know this?
1
Not at all
2
3
4
5
Perfectly
21
Q

___________________ knee ligaments are intraarticular.

A

Anterior and posterior cruciate

22
Q

One of your patients has decreased knee extension due to pain and a pinching feeling. He states that each time he straightens the leg out while sitting, it feels like something is being “pinched” under his kneecap. He likely has a _____________.

A

Patella plica

23
Q

Your 20 year old patient was injured while her foot was planted. She was diagnosed with grade three tears of the anterior cruciate ligament, medial collateral ligament and medial meniscus. This is known as_____________

A

The terrible triad

24
Q

What one action does the following muscles perform? The biceps femoris, semimembranosus, semitendinosus, and gastrocnemius

A

All flex the knee

25
Q

Abnormal patellar tracking can result in which of the following?

A

Chondromalacia patellae, osteoarthritis, or patellar dislocations

26
Q

With regard to the menisci of the knee, which statement is the MOST correct?

A

The outer edges can heal well, but the inner areas have poor to no blood supply

27
Q

With regards to the angle of inclination of the neck of the femur and the shaft of the femur, which statement is the most correct?

A

Less than 120 degrees is known as coxa vara

28
Q

As a PTA student you are tasked with watching your patient walk back and forth in the clinic so you can assess his gait. You notice that as he is walking he has a hip hike on the right side, which muscle is performing this action?

A

quadratus lumborum

29
Q

The ligamentum teres _________________________________.

A

encases the artery to the head of the femur

30
Q

The _____ is the projection of bone from the distal fibula.

A

Lateral malleolus

31
Q

Which two opposing muscles attach to the medial cuneiform?

A

Tibialis anterior and peroneus longus

32
Q

With regard to the sustentaculum tali, which statement is the MOST correct?

A

It is a projection of the calcaneus that supports the talus so it can support the tibia.

33
Q

https://image.shutterstock.com/image-photo/advanced-flat-feet-pes-planus-260nw-1779245159.jpg

A

Pes planus

34
Q

https://thetagoeclinic.co.uk/wp-content/uploads/2016/05/pes-300x188.jpg

A

Pes cavus

35
Q

The plantar calcaneonavicular or spring ligament:

A

helps to maintain the medial longitudinal arch of the foot, and by providing support to the head of the talus, bears the major portion of the body weight.

36
Q

A person who has genu varum is also likely to have ___________.

A

Coxa valga

37
Q

Where is the lesser trochanter found?

A

Distal, medial, and posterior to the greater trochanter on the femur

38
Q

Other than possibly the scapula, where is the largest sesamoid bone in the body found?

A

In the quadriceps tendon

39
Q

The calcaneal tuberosity_______________.

A

serves as the attachment site for the Achilles tendon

40
Q

The tibial and peroneal nerve are divisions of which larger nerve?

A

Sciatic

41
Q

Your patient has weakness of the quadricep muscles while standing in a closed chain position, you want to work on strengthening these muscles during your session, but you also teach the patient to use the ______________ to assist with knee extension

A

Soleus

42
Q

The tibialis posterior and the fibularis longus, both ______________________.

A

Muscles perform plantar flexion

43
Q

Most of the muscles which dorsiflex the ankle/foot are innervated by which nerve?

A

Deep fibular nerve

44
Q

Your patient has decreased ankle motion and your CI asks you which muscle courses on the medial side of the anterior-posterior axis of rotation and which motion does it produce. You state___________ and ___________.

A

tibialis posterior; inversion

45
Q

On your first day of your clinical rotation, you have a patient who states they are trying out for the track team at school. They are wanting to participate in hurdles and high jump. Since she has been practicing so much she has noticed pain in her foot when she “pushes off”. You watch her walk and notices she has pes planus. What is her likely condition?

A

plantar fasciitis

46
Q

Your patient has tight hamstrings, when standing with his knees fully extended you see _________________.

A

Posterior pelvic tilt

47
Q

You are wanting to isolate the gluteus maximus in order to muscle test. What do you ask your patient to do?

A

Bend your knee

48
Q

Your patient is in supine on the mat table with the legs hanging off, you notice the space between their low back and the mat increases, this is likely caused by ___________.

A

anterior pelvic tilt due to tight hip flexors

49
Q

You observe a patient with paraplegia standing with loftstrand crutches and lower limb braces leaning the pelvis and trunk posteriorly to the hip joints to maintain hip extension, this is known as ____________.

A

hanging on his “Y” ligaments

50
Q

Your patient has hip and knee flexion contractures, your CI asks you for an intervention that would be most beneficial. You state _________

A

getting the patient in prone